If Sukanya receives two e–mail messages from Lula, what is the maximum possible number of e–mail messages Sukanya rec...

Isabella on July 27, 2020

Why does this not work?

I cant figure out why the following would not work, where the HJ block is between the Ls JLHJLJ J is one of the first 3, the same is the last, and there exists an HJ block. Thank you!

Reply
Create a free account to read and take part in forum discussions.

Already have an account? log in

Skylar on July 27, 2020

@isabellaivanov, happy to help!

The game setup tells us that "Sukanya receives at least one and no more than two messages from each of them [H, J, and K]." However, in your proposed scenario, Sukanya receives 3 messages from J.

Does that make sense? Please let us know if you have any other questions!